0 Daumen
448 Aufrufe

Hallo :)

bei der folgenden Induktionsaufgabe komme ich leider nicht weiter:

zz: n> (1+(1/n))^n für n ≥3


Freue mich über eure Hilfe

Avatar von

1 Antwort

+1 Daumen
 
Beste Antwort

Aloha :)

Behauptung: \(\left(1+\frac{1}{n}\right)^n<n\quad;\quad n\ge3\)

Verankerung bei \(n=3\):$$\left(1+\frac{1}{n}\right)^n=\left(1+\frac{1}{3}\right)^3=\left(\frac{4}{3}\right)^3=\frac{64}{27}<\frac{81}{27}=3=n\quad\checkmark$$

Induktionsschritt \(n\to n+1\):

Betrachte zunächst:$$\left(1+\frac{1}{n+1}\right)-\left(1+\frac{1}{n}\right)=\frac{1}{n+1}-\frac{1}{n}=\frac{n-(n+1)}{n(n+1)}=-\frac{1}{n(n+1)}<0$$$$\Rightarrow\;\;\left(1+\frac{1}{n+1}\right)<\left(1+\frac{1}{n}\right)$$Damit ist nun:$$\left(1+\frac{1}{n+1}\right)^{n+1}<\left(1+\frac{1}{n}\right)^{n+1}=\underbrace{\left(1+\frac{1}{n}\right)^n}_{<n}\left(1+\frac{1}{n}\right)<n\cdot\frac{n+1}{n}=n+1$$

Avatar von 148 k 🚀

Vielen Dank <3

Sehr schöne Lösung, Tschaka gibt sich immer so viel Mühe beim Erklären. Dafür +1 von mir!!

Ein anderes Problem?

Stell deine Frage

Willkommen bei der Mathelounge! Stell deine Frage einfach und kostenlos

x
Made by a lovely community